Chronic Exam 4 GI & MS

Pataasin ang iyong marka sa homework at exams ngayon gamit ang Quizwiz!

The nurse is reviewing medications that can be used for female clients who have constipation-predominant irritable bowel syndrome (IBS). Which drugs are available for this health problem? (Select all that apply.) Select all that apply. a. Lubiprostone b. Cetuximab c. 5-fluorouracil d. Psyllium hydrophilic mucilloid e. Linaclotide

a. Lubiprostone d. Psyllium hydrophilic mucilloid e. Linaclotide Cetuximab and 5-fluorouracil are chemotherapeutic drugs used for clients who have colorectal cancer. The other drugs are available for female clients who have constipation-predominant IBS.

A nurse is administering sulfasalazine to a client. Which of the following data should the nurse collect to help identify an adverse drug reaction? (Select all that apply). A. Level of consciousness B. Skin integrity C. Temperature D. Urine output E. CBC

B, C, E

A nurse is caring for a client who has peptic ulcer disease. The nurse should monitor the client's phosphorus levels when administering which of the following drugs? A. Omeprazole B. Aluminum hydroxide C. Sucralfate D. Ranitidine

B. Aluminum hydroxide

A nurse should recognize that sulfasalazine is contraindicated for clients who have which of the following conditions? A. Pancreatitis B. Aspirin sensitivity C. Bronchitis D. GERD

B. Aspirin sensitivity

Which of these client assessment findings is typically associated with oral cancer? a. Dry sticky oral membranes b. Increased appetite c. Itchy rash in oral cavity d. Painless red or raised lesion

ANS: D A painless red or raised lesion often indicates a diagnosis of oral cancer. The client usually has a decreased appetite and thick secretions. Itchiness is not a common finding associated with oral cancer.

Which of the following drugs has protocols that require clients to meet specific risk-management criteria and sign a treatment agreement before the nurse can administer the drug? A. Lubiprostone B. Azathioprine C. Sulfasalazine D. Alosetron

D. Alosetron

A nurse is planning care for a client who is receiving chemotherapy and has a new prescription for ondansetron. Which of the following actions should the nurse plan to take? (Select all that apply). A. Infuse the drug 30 min prior to chemotherapy. B. Administer the drug when the client reports nausea. C. Infuse the drug slowly over 15 min. D. Administer the drug immediately following chemotherapy. E. Repeat the dose 4 hr after chemotherapy.

A, C, E

A nurse is planning teaching for a client who has been prescribed loperamide to treat diarrhea. Which of the following statements should the nurse plan to include? A. "Avoid driving or activities requiring alertness." B. "If you miss a dose, double the next dose." C. "Rinsing the mouth with alcohol-based mouthwash can reduce dryness." D. "Having one glass of wine each night can improve medication effectiveness."

A. "Avoid driving or activities requiring alertness."

A nurse is teaching a client who has a new prescription for sulfasalazine. Which of the following statements should the nurse make? A. "Use sunscreen and protective clothing while taking sulfasalazine to prevent sunburn." B. "The medication can stain your contact lenses green." C. "The medication can color your urine dark brown." D. "Take an iron supplement when you take sulfasalazine to prevent anemia."

A. "Use sunscreen and protective clothing while taking sulfasalazine to prevent sunburn."

A nurse is preparing to administer pantoprazole intravenously to prevent stress ulcers during surgery. What action(s) by the nurse is (are) most appropriate? (Select all that apply.) A: Administer the drug through a separate IV line. B: Infuse pantoprazole using an IV pump. C: Keep the drug in its original brown container. D: Take vital signs frequently during infusion. E: Use an in-line IV filter when infusing.

A: Administer the drug through a separate IV line. B: Infuse pantoprazole using an IV pump. E: Use an in-line IV filter when infusing. When infusing pantoprazole, use a separate IV line, a pump, and an in-line filter. A brown wrapper and frequent vital signs are not needed.

The nurse recalls that the risk factors for acute gastritis include which of the following? (Select all that apply.) A: Alcohol B: Caffeine C: Corticosteroids D: Fruit juice E: Nonsteroidal anti-inflammatory drugs (NSAIDs)

A: Alcohol B: Caffeine C: Corticosteroids E: Nonsteroidal anti-inflammatory drugs (NSAIDs) Risk factors for acute gastritis include alcohol, caffeine, corticosteroids, and chronic NSAID use. Fruit juice is not a risk factor, although in some people it does cause distress.

A nurse assesses a client who is prescribed alosetron. Which assessment question would the nurse ask this client before starting the drug? a. "Have you been experiencing any constipation?" b. "Are you eating a diet high in fiber and fluids?" c. "Do you have a history of high blood pressure?" d. "What vitamins and supplements are you taking?"

ANS: A Ischemic colitis is a life-threatening complication of alosetron. The nurse would assess the client for constipation because it places the client at risk for this complication. The other questions do not identify the risk for complications related to alosetron.

The nurse notes that the primary health care provider documented the presence of mucosal erythroplasia in a client. What does the nurse understand that this most likely means for this client? a. Early sign of oral cancer b. Fungal mouth infection c. Inflammation of the gums d. Obvious oral tumor

ANS: A Mucosal erythroplasia is the earliest sign of oral cancer. It is not a fungal infection, inflammation of the gums, or an obvious tumor.

A nurse is reviewing the medical record of a client who has a new prescription for ranitidine. The nurse should recognize that which of the following drugs interacts with ranitidine? A. Phenobarbital sodium B. Ketoconazole C. Lisinopril D. Hydrochlorothiazide

B. Ketoconazole

A nurse cares for a client who states, "My husband is repulsed by my colostomy and refuses to be intimate with me." How would the nurse respond? a. "Let's talk to the ostomy nurse to help you and your husband work through this." b. "You could try to wear longer lingerie that will better hide the ostomy appliance." c. "You should empty the pouch first so it will be less noticeable for your husband." d. "If you are not careful, you can hurt the stoma if you engage in sexual activity."

ANS: A The nurse would collaborate with the ostomy nurse to help the client and her husband work through intimacy issues. The nurse would not minimize the client's concern about her husband with ways to hide the ostomy. The client will not hurt the stoma by engaging in sexual activity.

A nurse cares for a client who had a colostomy placed in the ascending colon 2 weeks ago. The client states, "The stool in my pouch is still liquid." How would the nurse respond? a. "The stool will always be liquid with this type of colostomy." b. "Eating additional fiber will bulk up your stool and decrease diarrhea." c. "Your stool will become firmer over the next couple of weeks." d. "This is abnormal. I will contact your primary health care provider."

ANS: A The stool from an ascending colostomy can be expected to remain liquid because little large bowel is available to reabsorb the liquid from the stool. This finding is not abnormal. Liquid stool from an ascending colostomy will not become firmer with the addition of fiber to the client's diet or with the passage of time.

1.9. The nurse is caring for a client with peritonitis. What assessment findings would the nurse expect? (Select all that apply.) a. Nausea and vomiting b. Distended rigid abdomen c. Abdominal pain d. Bradycardia e. Decreased urinary output f. Fever

ANS: A, B, C, E, F a. Nausea and vomiting b. Distended rigid abdomen c. Abdominal pain e. Decreased urinary output f. Fever Peritonitis is an acute inflammatory disorder. Therefore, the client would likely have all of these signs and symptoms but would have tachycardia rather than bradycardia due to dehydration from fever.

1.6. After teaching a patient who has a permanent ileostomy, a nurse assesses the client's understanding. Which dietary items chosen for dinner indicate that the client needs further teaching? (Select all that apply.) a. Corn b. String beans c. Carrots d. Wheat rice e. Squash

ANS: A, B, D a. Corn b. String beans d. Wheat rice Clients with an ileostomy should be cautious of high-fiber and high-cellulose foods including corn, string beans, and rice. Carrots and squash are low-fiber items.

The nurse is caring for a client who is planning to have a laparoscopic colon resection for colorectal cancer tomorrow. Which statement by the client indicates a need for further teaching? a. "I should have less pain after this surgery compared to having a large incision." b. "I will probably be in the hospital for 3 to 4 days after surgery." c. "I will be able to walk around a little on the same day as the surgery." d. "I will be able to return to work in a week or two depending on how I do."

ANS: B All of these statements are correct about having minimally invasive laparoscopic surgery except that the hospital stay will likely be only 1 or 2 days.

The nurse is caring for a client who has been prescribed lubiprostone for irritable bowel syndrome (IBS-C). What health teaching will the nurse include about taking this drug? a. "This drug will make you very dry because it will decrease your diarrhea." b. "Be sure to take this drug with food and water to help manage constipation." c. "Avoid people who have infection as this drug will suppress your immune system." d. "Include high-fiber foods in your diet to help produce more solid stools."

ANS: B Lubiprostone is an oral laxative approved for women who have IBS with constipation (IBS-C). Water and food will also help to improve constipation. The drug is not used for clients who have diarrhea and does not affect the immune system. Although high-fiber foods are important for clients who have IBS, this client does not need fiber to help make stool more solid. Instead the fiber will help prevent constipation.

A nurse participates in a community screening event for oral cancer. What client is the highest priority for referral to a primary health care provider? a. Client who has poor oral hygiene practices. b. Client who smokes and drinks daily. c. Client who tans for an upcoming vacation. d. Client who occasionally uses illicit drugs.

ANS: B Smoking and alcohol exposure create a high risk for this client. Poor oral hygiene is not related to the etiology of cancer but may cause a tumor to go unnoticed. Tanning is a risk factor, but short-term exposure does not have the same risk as daily exposure to tobacco and alcohol.

A client is preparing to have a fecal occult blood test (FOBT). What health teaching would the nurse include prior to the test? a. "This test will determine whether you have colorectal cancer." b. "You need to avoid red meat and NSAIDs for 48 hours before the test." c. "You don't need to have this test because you can have a virtual colonoscopy." d. "This test can determine your genetic risk for developing colorectal cancer."

ANS: B The FOBT is a screening test that is sometimes used to assess for microscopic lower GI bleeding. To help prevent false positive results, the client needs to avoid red meat, Vitamin C, and NSAIDs. The test is not diagnostic nor does it determine a client's genetic risk for colorectal cancer.

The nurse is caring for a client diagnosed with oral cancer. What is the nurse's priority for client care? a. Encourage fluids to liquefy the client's secretions. b. Place the client on Aspiration Precautions. c. Remind the client to use an incentive spirometer. d. Manage the client's pain and inflammation.

ANS: B The client who has oral cancer often has difficulty swallowing and is at risk for aspiration and possibly aspiration pneumonia. Therefore, the most important nursing action is to place the client on precautions to prevent aspiration. The nurse would implement the other actions but they are not as vital to promote client safety.

The nurse is caring for a client with probable colorectal cancer (CRC). What assessment findings would the nurse expect? (Select all that apply.) a. Weight gain b. Rectal bleeding c. Anemia d. Change in stool shape e. Electrolyte imbalances f. Abdominal discomfort

ANS: B, C, D, F The client who has CRC usually experiences unintentional weight loss and rectal bleeding, either gross or occult. As a result of bleeding, the client has anemia and fatigue. Electrolyte imbalances are not common, but the client may note that the shape or consistency of stool has changed.

1.3. A nurse assesses a patient who has celiac disease. Which signs and symptoms would the nurse expect? (Select all that apply.) a. Weight gain b. Anorexia c. Constipation d. Anal fistula e. Abdominal pain

ANS: B, C, E b. Anorexia c. Constipation e. Abdominal pain Signs and symptoms of celiac disease include weight loss, anorexia, constipation, and abdominal pain. Anal fistulas are not associated with celiac disease.

1.7. A nurse cares for a patient who has a chronic inflammatory bowel disease. Which actions would the nurse take to prevent skin excoriation? (Select all that apply.) a. Cleanse the perineum with an antibacterial soap. b. Use medicated wipes instead of toilet paper. c. Identify foods that decrease constipation. d. Apply a thin coat of aloe cream to the perineum. e. Gently pat the perineum dry after cleansing.

ANS: B, D, E b. Use medicated wipes instead of toilet paper. d. Apply a thin coat of aloe cream to the perineum. e. Gently pat the perineum dry after cleansing. To prevent skin excoriation from frequent bowel movements associated with inflammatory bowel disease, the nurse would encourage good skin care with a mild soap and water and gently patting the area dry after each bowel movement. Using medicated wipes instead of toilet paper and applying a thin coat of aloe cream are appropriate. The client should identify and avoid foods that increase diarrhea. Antibacterial soaps are harsh and should not be used.

3. The nurse is caring for a client who is diagnosed with a complete small bowel obstruction. For what priority problem is this client most likely at risk? a. Abdominal distention b. Nausea c. Electrolyte imbalance d. Obstipation

ANS: C The client who has a small bowel obstruction is at the highest risk for fluid and electrolyte imbalances, especially dehydration and hypokalemia due to profuse vomiting. Nausea, abdominal distention, and obstipation are also usually present, but these problems are not as life threatening as the imbalances in electrolytes.

The nurse is teaching a client diagnosed with stomatitis about special mouth care. Which statement by the client indicates a need for further teaching? a. "I need to take out my dentures until my mouth heals." b. "I'll try to eat soft foods that aren't spicy and acidic." c. "I will use a more firm toothbrush to keep my mouth clean." d. "I'll be sure to rinse my mouth often with warm salt water."

ANS: C The client who has stomatitis has oral inflammation which causes discomfort. Therefore, all of these actions help to avoid irritation except for needing to use a soft toothbrush or gauze rather than a firm one.

A nurse is providing teaching to a client who has a new prescription for omeprazole to treat a duodenal ulcer. Which of the following instructions should the nurse include? A. Take the drug with food B. Swallow the capsules whole C. Dissolve the tablets in water D. Take the drug at bedtime

B. Swallow the capsules whole

After teaching a client who is recovering from a colon resection to treat early-stage colorectal cancer (CRC), the nurse assesses the client's understanding. Which statements by the client indicate understanding of the teaching? (Select all that apply.) a. "I must change the ostomy appliance daily and as needed." b. "I will use warm water and a soft washcloth to clean around the stoma." c. "I might start bicycling and swimming again once my incision has healed." d. "I will make sure that I make lifestyle changes to prevent constipation." e. "I will be sure to have the recommended colonoscopies."

ANS: C, D, E The client has had a colon resection for early CRC and there is no indication that the client also had a colostomy. Follow up with recommended colonoscopies are essential to monitor for CRC recurrence. Avoiding constipation will help improve intestinal motility which helps to decrease the risk for CRC recurrence. Exercise and other activities do not need to be restricted after the client has healed.

A nurse assesses clients at a community health center. Which client is at highest risk for developing colorectal cancer? a. A 37-year-old who drinks eight cups of coffee daily. b. A 44-year-old with irritable bowel syndrome (IBS). c. A 60-year-old lawyer who works 65 hours per week. d. A 72-year-old who eats fast food frequently.

ANS: D Colon cancer is rare before the age of 40, but its incidence increases rapidly with advancing age. Fast food tends to be high in fat and low in fiber, increasing the risk for colon cancer. Coffee intake, IBS, and a heavy workload do not increase the risk for colon cancer.

A nurse is providing teaching to a client who has a new prescription for loperamide. Which of the following instructions should the nurse include? A. Dissolve the powder thoroughly in 8 oz (237 mL) of water B. Take with diphenhydramine to prevent extrapyramidal effects C. Avoid activities that require alertness D. Take 30 minutes before activities that trigger nausea

C. Avoid activities that require alertness

The nurse is teaching a client about the use of viscous lidocaine for oral pain. What health teaching would the nurse include? a. "Use the drug before every meal to prevent aspiration." b. "Increase your intake of citrus foods to help with healing." c. "Use the drug only at bedtime because you won't be eating." d. "Be sure to check food temperatures before eating."

ANS: D Viscous lidocaine has an anesthetic effect in the oral cavity. Therefore, to promote client safety, the nurse would want to teach the client to check food temperature before eating.

A client who has peptic ulcer disease is prescribed quadruple drug therapy for Helicobacter pylori infection. What health teaching related to bismuth would the nurse include? A: "Report stool changes to your primary health care provider immediately." B: "Do not take aspirin or aspirin products of any kind while on bismuth." C: "Take bismuth about 30 minutes before each meal and at bedtime." D: "Be aware that bismuth can cause frequent vomiting and diarrhea."

B: "Do not take aspirin or aspirin products of any kind while on bismuth." Bismuth is a salicylate drug and causes stool discoloration but not vomiting and diarrhea. It does not have to be taken at a specific time relative to meals. Clients taking bismuth should not take other salicylates, such as aspirin or aspirin-containing products.

The nurse assesses a client who has possible gastritis. Which assessment finding(s) indicate(s) that the client has chronic gastritis? (Select all that apply.) A: Anorexia B: Dyspepsia C: Intolerance of fatty foods D: Pernicious anemia E: Nausea and vomiting

C: Intolerance of fatty foods D: Pernicious anemia Intolerance of fatty or spicy foods and pernicious anemia are signs of chronic gastritis. Anorexia and nausea/vomiting can be seen in both conditions. Dyspepsia is seen in acute gastritis.

A nurse is caring for a client who is taking phenytoin for a seizure disorder and has a new prescription for sucralfate to treat a duodenal ulcer. The nurse should instruct the client to take the drugs at least 2 hours apart for which of the following reasons? A. Phenytoin increases the metabolism of sucralfate. B. Phenytoin reduces the effectiveness of sucralfate. C. Sucralfate increases the risk for phenytoin toxicity. D. Sucralfate interferes with the absorption of phenytoin.

D. Sucralfate interferes with the absorption of phenytoin.

The nurse caring for clients with gastrointestinal disorders would recall that omeprazole is a drug in which classification? A: Gastric acid inhibitor B: Histamine receptor blocker C: Mucosal barrier fortifier D: Proton pump inhibitor

D: Proton pump inhibitor Omeprazole is a proton pump inhibitor.

16. The nurse assesses a patient who is recovering from an ileostomy placement. Which assessment finding would alert the nurse to immediately contact the primary health care provider? a. Pale and bluish stoma b. Liquid stool c. Ostomy pouch intact d. Blood-tinged output

a. Pale and bluish stoma The nurse would assess the stoma for color and contact the primary health care provider if the stoma is pale, bluish, or dark because these changes indicate possible lack of perfusion. The nurse would expect the client to have an intact ostomy pouch with dark green liquid stool that may contain some blood.

12. A nurse reviews the electronic health record of a client who has Crohn disease and a draining fistula. Which documentation would alert the nurse to urgently contact the primary health care provider for additional prescriptions? a. Serum potassium of 2.6 mEq/L (2.6 mmol/L) b. Client ate 20% of breakfast meal c. White blood cell count of 8200/mm3 (8.2 × 109/L) d. Client's weight decreased by 3 lb (1.4 kg)

a. Serum potassium of 2.6 mEq/L (2.6 mmol/L) Fistulas place the patient with Crohn disease at risk for hypokalemia which can lead to serious dysrhythmias. This potassium level is low and would cause the nurse to intervene. The white blood cell count is normal. The other two findings are abnormal and also warrant intervention, but the potassium level takes priority.

10. The nurse is caring for a client who is prescribed sulfasalazine. Which question would the nurse ask the client before starting this drug? a. "Are you taking Vitamin C or B? b. "Do you have any allergy to sulfa drugs?" c. "Can you swallow pills pretty easily?" d. "Do you have insurance to cover this drug?"

b. "Do you have any allergy to sulfa drugs?" Sulfasalazine is a sulfa drug given for clients who have ulcerative colitis. However, it should not be given to those who have an allergy to sulfa and sulfa drugs to prevent a hypersensitivity reaction.

The nurse is teaching a client with irritable bowel syndrome (IBS) who has frequent constipation. Which statement by the client shows an accurate understanding of the nurse's teaching? a. "Maintaining a low-fiber diet will manage my constipation." b. "I need to go for a walk every day if possible." c. "Limiting the amount of fluid that I drink with meals is very important." d. "A cup of caffeinated coffee with cream & sugar at dinner is OK for me."

b. "I need to go for a walk every day if possible." The client statement, "I need to go for a walk every evening," shows that the client accurately understands the nurse's teaching plan to treat IBS. Walking every day is an excellent exercise for promoting intestinal motility. Increased ambulation is part of the management plan for IBS, along with increased fluids and fiber and avoiding caffeinated beverages.

9. After teaching a client who is prescribed adalimumab for severe ulcerative colitis (UC), the nurse assesses the client's understanding. Which statement made by the client indicates a need for further teaching? a. "I will avoid large crowds and people who are sick." b. "I will take this medication with my breakfast each morning." c. "Nausea and vomiting are common side effects of this drug." d. "I should wash my hands after I play with my dog."

b. "I will take this medication with my breakfast each morning." Adalimumab is an immune modulator that is given via subcutaneous injection. It does not need to be given with food or milk. Nausea and vomiting are two common side effects. Adalimumab can cause immune suppression, so clients receiving the medication should avoid large crowds and people who are sick, and should practice good handwashing.

13. A client is preparing to have a laparoscopic restorative proctocolectomy with ileo pouch-anal anastomosis (RCA-IPAA). Which preoperative health teaching would the nurse include? a. "You will have to wear an appliance for your permanent ileostomy." b. "You should be able to have better bowel continence after healing occurs." c. "You will have a large abdominal incision that will require irrigation." d. "This procedure can be performed under general or regional anesthesia."

b. "You should be able to have better bowel continence after healing occurs." A RCA-IPAA can improve bowel continence although leakage may still occur for some clients. The procedure is a 2-step process performed under general anesthesia using a laparoscope which does not require an abdominal incision or permanent ileostomy.

A client has a nasogastric tube (NGT) connected to low continuous suction. What is the nurse's priority to ensure client safety? a. Assess for peristalsis at least once every 8 to 12 hours. b. Assess placement of the NGT for placement every 4 hours. c. Measure the gastric drainage every 8 to12 hours and document. d. Monitor the nasal skin and membranes around the tube for irritation.

b. Assess placement of the NGT for placement every 4 hours. Assessing the NGT for placement every 4 hours can help prevent aspiration which could lead to pneumonia. The other actions are appropriate for some clients, checking tube placement is the priority for care.

A client with a family history of colorectal cancer (CRC) regularly sees a primary health care provider for early detection of any signs of cancer. Which laboratory result may be an indication of CRC in this client? a. Decrease in liver function test results b. Elevated carcinoembryonic antigen c. Negative test for occult blood d. Elevated hemoglobin levels

b. Elevated carcinoembryonic antigen Carcinoembryonic antigen may be elevated in many patients diagnosed with CRC. Liver involvement may or may not occur in CRC. Hemoglobin will likely be decreased with CRC, not increased. An occult blood test is not reliable to affirm or rule out CRC.

15. The nurse plans care for a client with Crohn disease who has a heavily draining fistula. Which intervention would be the nurse's priority action? a. Low-fiber diet b. Skin protection c. Antibiotic administration d. Intravenous glucocorticoids

b. Skin protection Protecting the client's skin is the priority action for a patient who has a heavily draining fistula. Intestinal fluid enzymes are caustic and can cause skin breakdown or fungal infections if the skin is not protected. The plan of care for a client who has Crohn disease also includes adequate nutrition focused on high-calorie, high-protein, high-vitamin, and low-fiber meals, antibiotic administration, and glucocorticoids.

A client who has colorectal cancer is scheduled for a colostomy. Which referral is initially the most important for this client? a. Home health nursing agency b. Social worker c. Certified Wound, Ostomy, and Continence Nurse (CWOCN) d. Hospital chaplain

c. Certified Wound, Ostomy, and Continence Nurse (CWOCN) A CWOCN (or an enterostomal therapist) will be of greatest value to the client with colorectal cancer because the client is scheduled to receive a colostomy.The client is newly diagnosed, so it is not yet known whether home health nursing will be needed. A referral to hospice may be helpful for a terminally ill client. Referral to a chaplain may be helpful later in the process of adjusting to the disease.

5. The nurse assesses a client who is hospitalized with an exacerbation of Crohn disease. Which assessment finding would the nurse expect? a. Positive Murphy sign with rebound tenderness to palpitation b. Dull, hypoactive bowel sounds in the lower abdominal quadrants c. High-pitched, rushing bowel sounds in the right lower quadrant d. Reports of abdominal cramping that is worse at night

c. High-pitched, rushing bowel sounds in the right lower quadrant The nurse expects high-pitched, rushing bowel sounds due to narrowing of the bowel lumen in Crohn disease. A positive Murphy sign is indicative of gallbladder disease, and rebound tenderness often indicates peritonitis. Dullness in the lower abdominal quadrants and hypoactive bowel sounds is not commonly found with Crohn disease. Nightly worsening of abdominal cramping is not consistent with Crohn disease.

The nurse is caring for a client who had an anterior-posterior surgical resection for colorectal cancer this morning. What will the nurse anticipate as the client's priority problem at this time? a. Intestinal obstruction b. Nausea and vomiting c. Severe pain d. Constipation

c. Severe pain The surgical incisions are in the perineal area and are very painful due to the number of nerves in that region of the body. Pain control is the biggest challenge for the nurse and health care team to promote client comfort.

The nurse is teaching an older client how to prevent a stool impaction that can obstruct the intestines. Which statement by the client indicates a need for further teaching? a. "I will drink lots of fluids every day, especially water." b. "I will increase my exercise, especially walking, every day." c. "I will be sure to take a laxative every night to keep my bowels moving." d. "I will try to eat more high-fiber foods, such as raw vegetables and whole grains."

c."I will be sure to take a laxative every night to keep my bowels moving." All of these statements are correct except that the client should not take laxatives because they can decrease the tone of the abdominal muscles.

A nurse is caring for a client who has a new prescription for alosetron to treat irritable bowel syndrome. The nurse should instruct the client to report which of the following adverse effects of the drug? A. Headache B. Drowsiness C. Abdominal pain D. Sore throat

C. Abdominal pain

A client is admitted with a large oral tumor. What assessment by the nurse takes priority? a. Airway b. Breathing c. Circulation d. Nutrition

ANS: A Airway always takes priority. Airway must be assessed first and any problems managed if present.

A nurse cares for a client placed in skeletal traction. The client asks, "What is the primary purpose of this type of traction?" How would the nurse respond? a. "Skeletal traction will assist in realigning your fractured bone." b. "This treatment will prevent future complications and back pain." c. "Traction decreases muscle spasms that occur with a fracture." d. "This type of traction minimizes damage as a result of fracture treatment."

ANS: A Skeletal traction pins or screws are surgically inserted into the bone to aid in bone alignment. As a last resort, traction can be used to relieve pain, decrease muscle spasm, and prevent or correct deformity and tissue damage. These are not primary purposes of skeletal traction.

The nurse is teaching a client how to use a cane after a right surgical fractured fibula repair. What health teaching would the nurse include? a. "Place the cane on your left side." b. "Move the cane and your left leg at the same time." c. "Be sure the cane is parallel to your waist." d. "Use the cane only when your right leg is painful."

ANS: A The cane should be placed on the unaffected side (left for this client) and moved forward with the injured leg (right for this client) to provide support. The cane should be parallel to the stylus of the wrist and used at all times when ambulating.

A client is getting out of bed into the chair for the first time after an uncemented total hip arthroplasty. What action by the nurse is appropriate? a. Have adequate help to transfer the patient. b. Provide socks so the patient can slide easier. c. Tell the patient full weight bearing is allowed. d. Use a footstool to elevate the patient's leg.

ANS: A The client with an uncemented hip will be on toe-touch only after surgery. The nurse would ensure there is adequate help to transfer the patient while preventing falls. Slippery socks may cause a fall. Elevating the leg is not going to assist with the client's transfer.

The nurse assesses a patient who is recovering from an ileostomy placement. Which assessment finding would alert the nurse to immediately contact the primary health care provider? a. Pale and bluish stoma b. Liquid stool c. Ostomy pouch intact d. Blood-tinged output

ANS: A The nurse would assess the stoma for color and contact the primary health care provider if the stoma is pale, bluish, or dark because these changes indicate possible lack of perfusion. The nurse would expect the client to have an intact ostomy pouch with dark green liquid stool that may contain some blood

A nurse cares for a client who has a new colostomy. Which action would the nurse take? a. Empty the pouch frequently to remove excess gas collection. b. Change the ostomy pouch and barrier every morning. c. Allow the pouch to completely fill with stool prior to emptying it. d. Use surgical tape to secure the pouch and prevent leakage.

ANS: A The nurse would empty the new ostomy pouch frequently because of excess gas collection, and empty the pouch when it is one-third to one-half full of stool. The ostomy pouch does not need to be changed every morning. Ostomy barriers would be used to secure and seal the ostomy appliance; surgical tape would not be used.

The nurse is teaching a client about the risk of uncontrolled or untreated the client's gastroesophageal reflux disease (GERD). What complication(s) may occur if the GERD is not successfully managed? (Select all that apply.) a. Asthma b. Laryngitis c. Dental caries d. Cardiac disease e. Cancer

ANS: A, B, C, D, E Any of these complications may occur in clients who have uncontrolled or untreated GERD

The nurse is teaching assistive personnel about postoperative care for an older adult who had a posterolateral total hip arthroplasty. What teaching will the nurse include? (Select all that apply.) a. "Move the client slowly to prevent dizziness and a possible fall." b. "Encourage the client to deep breathe and cough at least every 2 hours." c. "Help the client use the incentive spirometer at least every 2 hours." d. "Keep the abduction pillow in place at all times while the client is in bed." e. "Let me know if the client has an elevated temperature or pulse." f. "Keep in mind that the client may be a little confused after surgery." g. "Please let me know if you see any reddened or open skin areas during bathing."

ANS: A, B, C, D, E, F Older adults are at risk for complications of decreased mobility after surgery, including atelectasis, pneumonia, pressure injuries, and orthostatic hypotension. Therefore these precautions are to help keep the client safe and avoid complications that could be life threatening.

The nurse is assessing a client with long-term rheumatoid arthritis (RA) who has been taking prednisone for 10 years. For which complications of chronic drug therapy would the nurse assess? (Select all that apply.) a. Osteoporosis b. Diabetes mellitus c. Glaucoma d. Hypertension e. Hypokalemia f. Decreased immunity

ANS: A, B, C, D, E, F Prednisone is a corticosteroid that is sometimes used for autoimmune disorders like RA when other drugs are not effective or cannot be tolerated. However, it can cause many complications when used long-term, including all of the health problems listed in the choices.

A client has rheumatoid arthritis (RA) and the nurse is conducting a home assessment. What options can the nurse suggest for the client to maintain independence in activities of daily living (ADLs)? (Select all that apply.) a. Grab bars to reach high items b. Long-handled bath scrub brush c. Soft rocker-recliner chair d. Toothbrush with built-up handle e. Wheelchair cushion for comfort

ANS: A, B, D Grab bars, long-handled bath brushes, and toothbrushes with built-up handles all provide modifications for daily activities, making it easier for the client with RA to complete ADLs independently. The rocker-recliner and wheelchair cushion are comfort measures but do not help increase independence. Most clients who have RA are not wheelchair-bound.

A nurse plans care for a client who is recovering from open reduction and internal fixation (ORIF) surgery for a right hip fracture. Which interventions would the nurse include in this client's plan of care? (Select all that apply.) a. Elevate heels off the bed with a pillow. b. Ambulate the client on the first postoperative day. c. Push the client's patient-controlled analgesia button. d. Re-position the client every 2 hours. e. Use pillows to encourage subluxation of the hip.

ANS: A, B, D Postoperative care for a client who has ORIF of the hip includes elevating the client's heels off the bed and re-positioning every 2 hours to prevent pressure and skin breakdown. It also includes ambulating the client on the first postoperative day, and using pillows or an abduction pillow to prevent subluxation of the hip. The nurse would teach the client to use the patient-controlled analgesia pump, but the nurse would never push the button for the client.

A nurse is visiting a client discharged home after a total hip arthroplasty. What safety precautions would the nurse recommend to the client and family? (Select all that apply.) a. Buy and install an elevated toilet seat. b. Install grab bars in the shower and by the toilet. c. Step into the bathtub with the affected leg first. d. Remove all throw rugs throughout the house. e. Use a shower chair while taking a shower.

ANS: A, B, D, E Buying and installing an elevated toilet seat, installing grab bars, removing throw rugs, and using a shower chair will all promote safety for this client. The client is still on partial weight bearing, so he or she cannot step into the bathtub leading with the operative side.

A nurse is assessing a community group for dietary factors that contribute to osteoporosis. In addition to inquiring about calcium, the nurse also assesses for which other dietary components? (Select all that apply.) a. Alcohol b. Caffeine c. Fat d. Carbonated beverages e. Vitamin D

ANS: A, B, D, E Dietary components that affect the development of osteoporosis include alcohol, caffeine, high phosphorus intake, carbonated beverages, and vitamin D. Tobacco is also a contributing lifestyle factor. Fat intake does not contribute to osteoporosis.

A client asks the nurse about what medications may be included for nonopioid multimodal analgesia following a total knee arthroplasty. What medications may be given to the client? (Select all that apply.) a. Gabapentin b. Ketorolac c. Hydrocodone d. Ketamine e. Morphine f. Bupivacaine

ANS: A, B, D, F All of the choices are appropriate to use for nonopioid multimodal analgesia except for the two opioid drugs—hydrocodone and morphine. The nonopioid medications are used to decrease inflammation and pain.

The nurse is assessing a client for chronic osteomyelitis. Which features distinguish this from the acute form of the disease? (Select all that apply.) a. Draining sinus tracts b. High fevers c. Presence of foot ulcers d. Swelling and redness e. Tenderness or pain

ANS: A, C Draining sinus tracts and foot ulcers are seen in chronic osteomyelitis. High fever, swelling, and redness are more often seen in acute osteomyelitis. Pain or tenderness can be in either case.

A nurse is planning postoperative care for a client following a total hip arthroplasty. What nursing interventions would help prevent venous thromboembolism for this client? (Select all that apply.) a. Early ambulation b. Fluid restriction c. Quadriceps-setting exercises d. Compression stockings/devices e. Anticoagulant drug therapy

ANS: A, C, D, E Early ambulation, leg exercises, and compression stockings/devices promote venous return and peripheral circulation which helps prevent deep vein thrombi. Anticoagulants such as subcutaneous low-molecular-weight heparin (LMWH) or factor Xa inhibitors are used for all clients who have a total lower extremity joint arthroplasty. The nurse would encourage fluids to expand blood volume and promote circulation; fluids would not be restricted.

A client with chronic osteomyelitis is being discharged from the hospital. What information is important for the nurse to teach this client and family? (Select all that apply.) a. Adherence to the antibiotic regimen b. Correct intramuscular injection technique c. Eating high-protein and high-carbohydrate foods d. Keeping daily follow-up appointments e. Proper use of the intravenous equipment

ANS: A, C, E The client going home with chronic osteomyelitis will need long-term antibiotic therapy—first intravenous, and then oral. The client needs education on how to properly administer IV antibiotics, care for the IV line, adhere to the regimen, and eat a healthy diet to encourage wound healing. The antibiotics are not given by IM injection. The client does not need daily follow-up.

A nurse is caring for a client who has a new prescription for ranitidine to treat GERD. The nurse should instruct the client to wait at least 1 hour between taking ranitidine and which of the following over-the-counter drugs? A. Ginkgo biloba B. Antidiarrheals C. St. John's wort D. Antacids

D. Antacids

The nurse assesses a client with diabetes and osteoarthritis (OA) during a checkup. The nurse notes the client's blood glucose readings have been elevated. What question by the nurse is most appropriate? a. "Are you following the prescribed diabetic diet?" b. "Have you been taking glucosamine supplements?" c. "How much exercise do you really get each week?" d. "You're still taking your diabetic medication, right?"

ANS: B All of the topics are appropriate for a client whose blood glucose readings have been higher than usual. However, since this client also has OA, and glucosamine can increase blood glucose levels, the nurse would ask about its use. The other questions all have an element of nontherapeutic communication in them. Asking how much exercise the client "really" gets is or if the diet is being followed is accusatory. Asking if the client takes his or her medications "right?" is patronizing.

A client is admitted to the emergency department with a fractured femur resulting from a motor vehicle crash. What the nurse's priority action? a. Keep the client warm and comfortable. b. Assess airway, breathing, and circulation. c. Maintain the client in a supine position. d. Immobilize the injured extremity with a splint.

ANS: B As part of the primary survey, the nurse would ensure that the client does not have any life- threatening problem by assessing the ABCs first. If there are not major problems, then the nurse could attend to the injured extremity.

A client with osteoporosis is going home where the client lives alone. What action by the nurse is best? a. Refer the client to Meals on Wheels. b. Arrange a home safety evaluation. c. Ensure that the client has a walker at home. d. Help the client look into assisted living.

ANS: B This client has several risk factors that place him or her at a high risk for falling. The nurse should consult social work or home health care to conduct a home safety evaluation. The other options may or may not be needed based upon the client's condition at discharge.

The nurse is caring for a client who had an open traditional esophagectomy. Which assessment findings would the nurse report immediately to the primary health care provider? (Select all that apply.) a. Nausea b. Wound dehiscence c. Fever d. Tachycardia e. Moderate pain f. Fatigue

ANS: B, C, D Wound dehiscence is a serious, potentially life-threatening problem that needs immediate attention of the primary health care provider, typically the surgeon. Fever and tachycardia may indicate that the client has a postoperative infection, another serious, potentially life-threatening complication. Indications of both of these problems need to be documented and reported by the nurse. Nausea, fatigue, and moderate pain are expected postoperative assessment findings.

The nurse assesses a client who is admitted with a pelvic fracture. Which assessments would the nurse monitor to prevent or detect a complication of this injury? (Select all that apply.) a. Temperature b. Urinary output c. Blood pressure d. Pupil reaction e. Skin color

ANS: B, C, E With a pelvic fracture, internal organ damage may result in bleeding and hypovolemic shock. The nurse monitors the client's heart rate, blood pressure, urine output, skin color, and level of consciousness frequently to detect assess for shock. It is important to monitor the urine for blood to assess whether the urinary system has been damaged with the pelvic fracture. Changes in temperature and pupil reactions are not directly associated with hypovolemic shock. Temperature changes are usually associated with hypo- or hyperthermia or infectious processes. Pupillary changes occur with brain injuries, bleeds, or strokes.

A nurse cares for a patient who has a chronic inflammatory bowel disease. Which actions would the nurse take to prevent skin excoriation? (Select all that apply.) a. Cleanse the perineum with an antibacterial soap. b. Use medicated wipes instead of toilet paper. c. Identify foods that decrease constipation. d. Apply a thin coat of aloe cream to the perineum. e. Gently pat the perineum dry after cleansing.

ANS: B, D, E To prevent skin excoriation from frequent bowel movements associated with inflammatory bowel disease, the nurse would encourage good skin care with a mild soap and water and gently patting the area dry after each bowel movement. Using medicated wipes instead of toilet paper and applying a thin coat of aloe cream are appropriate. The client should identify and avoid foods that increase diarrhea. Antibacterial soaps are harsh and should not be used.

A nurse is providing teaching to a client who has a new prescription for dimenhydrinate to prevent motion sickness. Which of the following instructions should the nurse include? (Select all that apply). A. Sit upright for 30 minutes after taking the drug B. Avoid antacids C. Take the drug 30 to 60 minutes before activities that trigger nausea D. Avoid activities that require alertness E. Increase fluid and fiber intake

C, D, E

An older client with diabetes is admitted with a heavily draining leg wound. The client's white blood cell count is 38,000/mm3 (38 x 10^9/L) but the client is afebrile. Which nursing action is most appropriate at this time? a. Administer acetaminophen as needed. b. Educate the client on amputation. c. Place the client on Contact Precautions. d. Refer the client to the wound care nurse.

ANS: C In the presence of a heavily draining wound, the nurse would place the client on Contact Precautions. If the client has discomfort, acetaminophen can be used, but this client has not reported pain and is afebrile. The client may or may not need an amputation in the future. The wound care nurse may be consulted but not as the most appropriate action.

The primary health care provider prescribes methotrexate (MTX) for a client with a new diagnosis of rheumatoid arthritis. The nurse provides health teaching about the drug. What statement by the nurse is appropriate to include about methotrexate? a. "It will take at least 1 to 2 weeks for the drug to help relieve your symptoms." b. "The drug is very expensive but there are pharmacy plans to help pay for it." c. "The drug can increase your risk for infection, so you should avoid crowds." d. "It's OK for you to drink about 2 to 3 glasses of wine each week while taking the drug."

ANS: C MTX takes up to 4 to 6 weeks to begin to help relieve RA symptoms and is very inexpensive. Clients should avoid alcohol due to the potential for liver toxicity. MTX suppresses the immune system which makes clients susceptible to infection. The nurse teaches clients to avoid crowds and anyone with a known infection.

A client had a bunionectomy with osteotomy. The client asks why healing may take up to 3 months. What explanation by the nurse is best? a. "The bones in your feet are hard to operate on." b. "The surrounding bones and tissue are damaged." c. "Your feet have less blood flow, so healing is slower." d. "Your feet bear weight so they never really heal."

ANS: C The feet are the most distal to the heart and receive less blood flow than other organs and tissues, prolonging the healing time after surgery. The other explanations are not correct.

5. A nurse assessing a client with colorectal cancer auscultates high-pitched bowel sounds and notes the presence of visible peristaltic waves. Which action would the nurse take? a. Ask if the client is experiencing pain in the right shoulder. b. Perform a rectal examination and assess for polyps. c. Recommend that the client have computed tomography. d. Administer a laxative to increase bowel movement activity.

ANS: C The presence of visible peristaltic waves, accompanied by high-pitched or high-pitched bowel sounds, is indicative of bowel obstruction caused by the tumor. The nurse would contact the primary health care provider with these results and recommend a computed tomography scan for further diagnostic testing. This assessment finding is not associated with right shoulder pain; peritonitis and cholecystitis are associated with referred pain to the right shoulder. The nurse generalist is not qualified to complete a rectal examination for polyps, and laxatives would not help this client.

The nurse is caring for a client who has severe osteoarthritis. What primary joint problems will the nurse expect the client to report? a. Crepitus b. Effusions c. Pain d. Deformities

ANS: C The primary assessment finding typically reported by clients who have osteoarthritis is joint pain, although crepitus, effusions (fluid), and mild deformities may occur.

The nurse is caring for several clients with osteoporosis. For which client would bisphosphonates not be a good option? a. Client with diabetes who has a serum creatinine of 0.8 mg/dL (61 mcmol/L). b. Client who recently fell and has vertebral compression fractures. c. Hypertensive client who takes calcium channel blockers. d. Client with a spinal cord injury who cannot tolerate sitting up.

ANS: D Clients on bisphosphonates must be able to sit upright for 30 to 60 minutes after taking them. The client who cannot tolerate sitting up is not a good candidate for this class of drug. Poor renal function also makes clients poor candidates for this drug, but the client with a creatinine of 0.8 mg/dL (61 mcmol/L) is within normal range. Diabetes and hypertension are not related unless the client also has renal disease. The client who recently fell and sustained fractures is a good candidate for this drug if the fractures are related to osteoporosis.

A nurse is caring for several clients with fractures. Which client would the nurse identify as being at the highest risk for developing deep vein thrombosis? a. An 18-year-old male athlete with a fractured clavicle b. A 36-year-old female with type 2 diabetes and fractured ribs c. A 55-year-old female prescribed ibuprofen for osteoarthritis d. A 74-year-old male who smokes and has a fractured pelvis

ANS: D Deep vein thrombosis (DVT) as a complication with bone fractures occurs more often when fractures are sustained in the lower extremities and the client has additional risk factors for thrombus formation. Other risk factors include obesity, smoking, oral contraceptives, previous thrombus events, advanced age, venous stasis, and heart disease. The other clients do not have additional risk factors for DVT.

A nurse cares for a client who had a wrist cast applied 3 days ago. The client states, "The cast is loose enough to slide off." How would the nurse respond? a. "Keep your arm above the level of your heart." b. "As your muscles atrophy, the cast is expected to loosen." c. "I will wrap a bandage around the cast to prevent it from slipping." d. "You need a new cast now that the swelling is decreased."

ANS: D Often the surrounding soft tissues may be swollen considerably when the cast is initially applied. After the swelling has resolved, if the cast is loose enough to permit two or more fingers between the cast and the client's skin, the cast needs to be replaced. Elevating the arm will not solve the problem, and the client's muscles should not atrophy while in a cast for 6 weeks or less. An elastic bandage will not prevent slippage of the cast.

The nurse is caring for a young client who has been diagnosed with osteopenia. Which risk factor in the client's history most likely contributed to the bone loss? a. Osteoarthritis b. Hypothyroidism c. Addison disease d. Rheumatoid arthritis

ANS: D Rheumatoid arthritis often occurs in young female adults and can lead to osteoporosis as a common complication. Cushing disease (rather than Addison disease) and hyperthyroidism (rather than hypothyroidism) are also risk factors. Osteoarthritis is a joint disease.

The nurse is caring for a client who has been diagnosed with peptic ulcer disease. For which complication would the nurse monitor? A: Large bowel obstruction B: Dyspepsia C: Upper gastrointestinal (GI) bleeding D: Gastric cancer

C: Upper gastrointestinal (GI) bleeding Peptic ulcer disease (PUD) can cause gastric mucosal damage or perforation, which causes upper GI bleeding. Dyspepsia is a symptom of PUD, gastritis, and gastric cancer. PUD affects the stomach and/or duodenum, not the colon.

The nurse is teaching a client who has been treated for acute gastritis. What statement by the client indicates a need for further teaching? A: "I need to cut down on drinking martinis every might." B: "I should decrease my intake of caffeinated drinks, especially coffee." C: "I will only take ibuprofen once in a while when I really need it." D: "I can continue smoking cigarettes which is better than chewing tobacco."

D: "I can continue smoking cigarettes which is better than chewing tobacco." To prevent another episode of acute gastritis, alcohol, caffeinated drinks, and NSAIDs should be avoided or kept at a minimum. Smoking and all forms of tobacco should also be avoided.

The nurse is caring for a client experiencing upper gastrointestinal (GI) bleeding. What is the priority action for the client's care? A: Maintain airway, breathing, and circulation. B: Monitor vital signs, including orthostatic blood pressures. C: Draw blood for hemoglobin and hematocrit immediately. D: Insert a nasogastric (NG) tube and connect to intermittent suction.

A: Maintain airway, breathing, and circulation. The priority action for any client experiencing deterioration or an emergent situation is monitor and maintain airway, breathing, and circulation (ABCs). Taking orthostatic blood pressures would not be appropriate, but the nurse would monitor vital signs carefully and draw blood for hemoglobin and hematocrit. An NG tube would also need to be inserted and connected to gastric suction to rest the GI tract. However, none of these actions take priority over maintaining ABCs.

A nurse is caring for a client with diabetes mellitus who has fractured her arm. Which action would the nurse take first? a. Remove the medical alert bracelet from the fractured arm. b. Immobilize the arm by splinting the fractured site. c. Place the client in a supine position with a warm blanket. d. Cover any open areas with a sterile dressing.

ANS: A A client's medical alert bracelet or any other jewelry would be removed from the fractured arm before the affected extremity swells. Immobilization, positioning, and dressing should occur after the bracelet is removed.

The nurse is teaching a client who is prescribed acetaminophen for control of osteoarthritic joint pain. What statement by the client indicates a need for further teaching? a. "I won't take more than 5000 mg of this drug each day." b. "I'll follow up to get my lab tests done to check my liver." c. "I'll check drugs that I take for acetaminophen in them." d. "I can use topical patches and creams to help relieve pain."

ANS: A All of the choices are correct about acetaminophen except that the maximum daily dosage is 4000 mg. For older adults, 3000 mg are recommended due to slower drug metabolism by the liver.

A nurse is teaching a client newly diagnosed with osteoarthritis (OA) about drugs used to treat the disease. For which drug does the nurse plan health teaching? a. Acetaminophen b. Cyclobenzaprine hydrochloride c. Hyaluronate d. Ibuprofen

ANS: A All of these drugs may be appropriate to treat OA. However, the first-line drug is acetaminophen. Cyclobenzaprine is a muscle relaxant given to treat muscle spasms. Hyaluronate is a synthetic joint fluid implant. Ibuprofen is a nonsteroidal anti-inflammatory drug.

After teaching a client who is recovering from a vertebroplasty, the nurse assesses the patient's understanding. Which statement by the client indicates a need for additional teaching? a. "I can drive myself home after the procedure." b. "I will monitor the puncture site for signs of infection." c. "I can start walking tomorrow and increase my activity slowly." d. "I will remove the dressing the day after discharge."

ANS: A Before discharge, a client who has a vertebroplasty would be taught to avoid driving or operating machinery for the first 24 hours. The client should monitor the puncture site for signs of infection. Usual activities can resume slowly, including walking and slowly increasing activity over the next few days. The client should keep the dressing dry and remove it the next day.

A client who has rheumatoid arthritis is prescribed etanercept. What health teaching by the nurse about this drug is appropriate? a. Giving subcutaneous injections b. Having a chest x-ray once a year c. Taking the medication with food d. Using heat on the injection site

ANS: A Etanercept is given as a subcutaneous injection twice a week. The nurse would teach the client how to self-administer the medication. The other options are not appropriate for etanercept.

A client has been advised to perform weight-bearing exercises to help slow bone loss, but has not followed this advice. What response by the nurse is appropriate at this time? a. Ask the client about fear of falling. b. Instruct the client to increase calcium. c. Suggest other exercises the client can do. d. Tell the client to try weight lifting.

ANS: A Fear of falling can limit participation in activity. The nurse would first assess if the client has this fear and then offer suggestions for dealing with it. The client may or may not need extra calcium, other exercises, or weight lifting.

The nurse is caring for a client with sialadenitis. What comfort measures are appropriate for this client? (Select all that apply.) a. Applying warm compresses b. Applying ice to salivary glands c. Offering fluids every hour d. Providing lemon-glycerin swabs e. Reminding the patient to avoid speaking

ANS: A, C Warm compresses and fluids can help promote comfort for this client. Application of ice or lemon-glycerin swabs would not be used. Speaking has no effect on this condition.

A client who had a surgical fractured femur repair reports new-onset shortness of breath and increased respirations. What is the nurse's first action? a. Place the client in a high-Fowler position. b. Document the client's oxygen saturation level. c. Start oxygen therapy at 2 L/min via nasal cannula. d. Contact the primary health care provider.

ANS: A The client is experiencing respiratory distress which could be due to pulmonary embolus, fat embolism syndrome, or anxiety. Regardless of the cause, the nurse would place the client in a sitting position first and then perform additional assessment. Oxygen would likely be needed, especially if the client's oxygen saturation was under 95%.

A client with bone cancer is hospitalized for a limb salvage procedure. How can the nurse best address the client's psychosocial needs? a. Assess the client's coping skills and support systems. b. Explain that the surgery leads to a longer life expectancy. c. Refer the client to the social worker or hospital chaplain. d. Reinforce physical therapy to aid with ambulating normally.

ANS: A The first step in the nursing process is assessment. The nurse would assess coping skills and possible support systems that will be helpful in this client's treatment. Explaining that a limb salvage procedure will extend life does not address the client's psychosocial needs. Referrals may be necessary, but the nurse should assess first. Reinforcing physical therapy is also helpful but again does not address the psychosocial needs of the client.

The nurse assesses a client after a total hip arthroplasty. The client's surgical leg is visibly shorter than the other one and the client reports extreme pain. While a co-worker calls the surgeon, what action by the nurse is appropriate? a. Assess neurovascular status in both legs. b. Elevate the surgical leg and apply ice. c. Prepare to administer pain medication. d. Try to place the surgical leg in abduction.

ANS: A This client has signs and symptoms of hip dislocation, a potential complication of this surgery. Hip dislocation can cause neurovascular compromise. The nurse would assess neurovascular status while comparing both legs. The nurse would not try to move the extremity to elevate or abduct it. Pain medication may be administered if possible, but first the nurse would thoroughly assess the client. DIF: Applying TOP: Integrated Process: Nursing Process: Assessment KEY: Total joint arthroplasty, Perioperative care MSC: Client Needs Category: Physiological Integrity: Reduction of Risk Potential

What information does the nurse teach a women's group about osteoporosis? a. "Primary osteoporosis occurs in postmenopausal women due to lack of estrogen." b. "Men actually have higher rates of the disease but are underdiagnosed." c. "There is no way to prevent or slow osteoporosis after menopause." d. "Women and men have an equal chance of getting osteoporosis."

ANS: A Women are more at risk of developing primary osteoporosis after menopause due to the lack of estrogen. Men have a slower loss of bone after the age of 75. Many treatments are now available for women to slow osteoporosis after menopause.

1.8. The nurse is caring for a client who is diagnosed with celiac disease and preparing to start natalizumab. Which health teaching would the nurse include in the teaching? (Select all that apply.) a. Need to have drug administered by a primary health care provider. b. Need to avoid crowds and individuals who have infection. c. Need to report injection reactions such as redness and swelling. d. Awareness of a rare but potentially fatal drug complication. e. Need to report any signs and symptoms of infection immediately.

ANS: A, B, D, E a. Need to have drug administered by a primary health care provider. b. Need to avoid crowds and individuals who have infection. d. Awareness of a rare but potentially fatal drug complication. e. Need to report any signs and symptoms of infection immediately. All of these choices are correct except that the drug is given intravenously. Therefore, there is no need to teach the client to report injection reactions because the client does not self-administer the medication subcutaneously. Natalizumab can cause progressive multifocal leukoencephalopathy (PML), but it is a very rare disorder causing cognitive, sensory, and/or motor changes.

A nurse teaches a client with a fractured tibia about external fixation. Which advantages of external fixation for the immobilization of fractures would the nurse share with the client? (Select all that apply.) a. It leads to minimal blood loss. b. It allows for early ambulation. c. It decreases the risk of infection. d. It increases blood supply to tissues. e. It promotes healing.

ANS: A, B, E External fixation is a system in which pins or wires are inserted through the skin and bone and then connected to a ridged external frame. With external fixation, blood loss is less than with internal fixation, but the risk for infection is much higher. The device allows early ambulation and exercise, maintains alignment, stabilizes the fracture site, and promotes healing. The device does not increase blood supply to the tissues. The nurse would assess for distal circulation, movement, and sensation, which can be disturbed by fracture injuries and treatments.

The nurse is caring for an older client who had a total knee arthroplasty. Prior to surgery, the client lived alone independently. With which interprofessional health care team members will the nurse collaborate to ensure positive client outcomes? (Select all that apply.) a. Case manager b. Mental health counselor c. Physical therapist d. Occupational therapist e. Speech-language pathologist f. Clergy/Spiritual leader

ANS: A, C The client was independent and living alone prior to surgery but will likely need help for a short time at home. However, if the client was ADL independent, he or she will not need referral to an occupational therapist. Therefore, a case manager can assess the living situation and identify any special needs to be addressed. The physical therapist will help the client learn to ambulate independently with a walker. There is no indication that the client needs referral for mental, spiritual, or speech-language services.

A client has a bone density score of -2.8. What intervention would the nurse anticipate based on this assessment? a. Asking the client to complete a food diary b. Planning to teach about bisphosphonates c. Scheduling another scan in 2 years d. Scheduling another scan in 6 months

ANS: B A T-score from a bone density scan at or lower than -2.5 indicates osteoporosis. The nurse would plan to teach about medications used to treat this disease, such as the bisphosphonates. A food diary is helpful to determine if the client gets adequate calcium and vitamin D, but at this point, dietary changes will not prevent the disease. Simply scheduling another scan will not help treat the disease either.

A nurse assesses a group of clients who have rheumatoid arthritis (RA). Which client would the nurse see first? a. Client who reports jaw pain when eating b. Client with a red, hot, swollen right wrist c. Client who has a puffy-looking area behind the knee d. Client with a worse joint deformity since the last visit

ANS: B All of the options are possible manifestations of RA. However, the presence of one joint that is much redder, hotter, or more swollen that the other joints may indicate infection or an exacerbation of the RA disease process. The nurse needs to see this client first.

A client with rheumatoid arthritis (RA) has an acutely swollen, red, and painful joint. What nonpharmacologic intervention does the nurse recommend? a. Heating pad b. Ice packs c. Splint d. Paraffin dip

ANS: B Ice is best for acute inflammation. Heat often helps with joint stiffness. Splinting helps preserve joint function. A paraffin dip is used to provide warmth to the joint which is more appropriate for chronic pain and stiffness.

A client asks the nurse about having a total knee arthroplasty to relieve joint pain. Which factor would place the client at the highest risk for impaired postoperative healing? a. Controlled hypertension b. Obesity c. Osteoarthritis d. Mild osteopenia

ANS: B Obesity places a client at high risk for many postoperative complications including slower wound and bone healing. The other factors usually do not affect healing after surgery.

A client is prescribed celecoxib for joint pain. What statement by the client indicates a need for further teaching? a. "I'll report any signs of bleeding or bruising to my primary health care provider." b. "I'll take this drug only as prescribed by my primary health care provider." c. "I'll be sure to take this drug three times a day only on an empty stomach." d. "I'll monitor the amount of urine that I excrete every day and report any changes."

ANS: C All of the choices are correct for this NSAID except that celecoxib can cause GI distress unless taken with meals or food. The drug should not be taken on an empty stomach and is rarely taken more than twice a day.

A nurse cares for a client with a recently fractured tibia. Which assessment would alert the nurse to take immediate action? a. Pain of 4 on a scale of 0-10 b. Numbness in the extremity c. Swollen extremity at the injury site d. Feeling cold while lying in bed

ANS: B The client with numbness and/or tingling of the extremity may be displaying the first signs of acute compartment syndrome. This is an acute problem that requires immediate intervention because of possible decreased circulation. Moderate pain and swelling is an expected assessment after a fracture. These findings can be treated with comfort measures. Being cold can be treated with additional blankets or by increasing the temperature of the room.

The nurse is performing a neurovascular assessment for an older client who has an extremity fracture. How many seconds would the nurse expect for a capillary refill in it is within normal range? a. 20 seconds b. 15 seconds c. 10 seconds d. 5 seconds

ANS: D The normal capillary refill is usually 3 seconds, but for older adults, the refill usually takes up to 5 seconds due to vascular changes associated with aging.

The nursing is teaching a client diagnosed with gastroesophageal reflux disease (GERD) who is planning to have an endoscopic radiofrequency (Stretta) procedure. What preprocedure health teaching would the nurse include? (Select all that apply.) a. "You will need to be on a liquid diet for the first week after the procedure." b. "Avoid taking any NSAIDs like ibuprofen for 10 days before the procedure." c. "Contact the primary health care provider after the procedure if you have increased pain." d. "You will need a nasogastric tube for a few days after the procedure." e. "You will have a small incision in your stomach area that will have a wound closure.

ANS: B, C The client having this procedure does not have an incision and will not require a nasogastric tube (NGT). The client should avoid an NGT placement for at least a month after the procedure. A liquid diet is required for only 24 hours after the procedure and then the client should progress to include soft floods like custard and applesauce.

The nurse is teaching a client who had a descending colostomy 2 days ago about the ostomy stoma. Which changes in the stoma would the nurse teach the client to report to the primary health care provider? (Select all that apply.) a. Stool consistency is similar to paste. b. Stoma becomes dark and dull. c. Skin around the stoma becomes excoriated. d. Skin around stoma becomes protruded. e. Stoma becomes retracted into the abdomen.

ANS: B, C, D, E A colostomy placed in the descending colon would be expect to have a paste-like stool consistency. However, if the stoma becomes retracted or discolored, the client should report those changes to the primary health care provider. Skin around the stoma that becomes protruded would suggest the formation of a peristomal hernia, and skin excoriation needs appropriate management. Therefore, both of those skin changes would need to be reported to the primary health care provider.

The nurse is caring for a client who just had a kyphoplasty. What nursing care is needed for the client at this time? (Select all that apply.) a. Place the client in a prone position to prevent pressure on the surgical area. b. Apply an ice pack to the surgical area to help relieve pain. c. Assess the client's pain level to compare it with pain before the procedure. d. Take vital signs, including oxygen saturation, frequently. e. Monitor for complications such as bleeding or shortness of breath. f. Perform frequent neurologic assessments and report major changes.

ANS: B, C, D, E, F All of the choices are correct except that the client should stay in a flat supine position immediately after the procedure.

The nurse is teaching a client with mild rheumatoid arthritis (RA) about how to protect synovial joints. Which health teaching will the nurse include? (Select all that apply.) a. "Use small joints rather than larger ones during tasks." b. "Use both hands instead of one with holding objects." c. "When getting out of bed or a chair, use the palms of your hands." d. "Bend your knees instead of your waist and keep your back straight." e. "Do not use multiple pillows under your head to prevent neck flexion." f. "Use a device or rubber grip to open jars or bottle tops." g. "Use long-handled devices such as a hairbrush with an extended handle."

ANS: B, C, D, E, F, G All of these options are part of health teaching for joint protection except that large joints should be used instead of smaller ones.

The nurse assesses a client with long-term rheumatoid arthritis (RA) for late signs and symptoms. Which assessment findings will the nurse document as late signs and symptoms of RA? (Select all that apply.) a. Anorexia b. Felty syndrome c. Joint deformity d. Low-grade fever e. Weight loss

ANS: B, C, E Late signs and symptoms of RA include Felty syndrome, joint deformity, weight loss, organ involvement, osteoporosis, extreme fatigue, and anemia, among others. Anorexia and low-grade fever are both seen early in the course of the disease.

The nurse is teaching a client who is planning to have a total hip arthroplasty. What statement by the client indicates a need for further teaching? a. "I will get an IV antibiotic right before surgery to prevent infection." b. "I may request a regional nerve block as part of the surgical anesthesia." c. "I will receive IV heparin before surgery to decrease the risk of clots." d. "I will receive tranexamic acid to help reduce blood loss during surgery."

ANS: C All of the choices are correct except that IV heparin is not given before or after surgery. A different anticoagulant is given after surgery to prevent postoperative venous thromboembolism, such as deep vein thrombosis and pulmonary embolus.

A client has a left knee arthrocentesis to remove excess joint fluid. What post procedure health teaching will the nurse include? a. "Take your opioid medication as prescribed by the primary health care provider." b. "Do not bear weight on your left leg for at least a week after you get home." c. "Monitor the site for bleeding or clear fluid leakage when you are home." d. "Tell your employer that you can't come back to work for 2 to 3 weeks."

ANS: C An arthrocentesis is performed as an ambulatory procedure and may require a mild analgesic such as acetaminophen for discomfort. Opioids are not used. The client may bear weight and return to work, but needs to monitor for bleeding or leakage of synovial fluid at the injection site.

The nurse teaches assistive personnel (AP) about care of an older adult diagnosed with osteoporosis. What teaching would the nurse include? a. "Teach the client to eat high-calcium foods in the diet." b. "Assist the client with activities of daily living." c. "Osteoporosis places the client is at risk for fractures." d. "The client should stay in bed to prevent falling."

ANS: C Anyone who has osteoporosis is at risk for fragility fractures even if he or she does not experience trauma like a fall. The client needs to keep active rather than stay in bed where more bone could be lost. High-calcium foods may not be helpful because bone loss is already severe. There is no indication that the client needs assistance with ADLs.

A nurse cares for a client with colorectal cancer who has a new colostomy. The client states, "I think it would be helpful to talk with someone who has had a similar experience." How would the nurse respond? a. "I have a good friend with a colostomy who would be willing to talk with you." b. "The ostomy nurse will be able to answer all of your questions." c. "I will make a referral to the United Ostomy Associations of America." d. "You'll find that most people with colostomies don't want to talk about them."

ANS: C Nurses need to become familiar with community-based resources to better assist clients. The local chapter of the United Ostomy Associations of America has resources for clients and their families, including ostomates (specially trained visitors who also have ostomies). The nurse would not suggest that the client speak with a personal contact of the nurse. Although the ostomy nurse is an expert in ostomy care, talking with him or her is not the same as talking with someone who actually has had a colostomy. The nurse would not brush aside the client's request by saying that most people with colostomies do not want to talk about them. Many people are willing to share their ostomy experience in the hope of helping others.

A client has long-term rheumatoid arthritis that especially affects the hands. The client wants to finish quilting a baby blanket before the birth of her grandchild. What response by the nurse is appropriate? a. "Let's ask your provider about increasing your pain pills." b. "Hold ice bags against your hands before quilting." c. "Try a paraffin wax dip 20 minutes before you quilt." d. "You need to stop quilting before it destroys your fingers."

ANS: C Paraffin wax dips are beneficial for decreasing pain in arthritic hands and lead to increased mobility. Increasing pain pills may not help with movement. Ice has limited use unless the client has a "hot" or exacerbated joint. The client wants to finish the project, so the nurse would not negate its importance by telling the client it is destroying her joints.

A nurse assesses clients in an osteoporosis clinic. Which client would the nurse assess first? a. Client taking calcium with vitamin D who reports flank pain 2 weeks ago. b. Client taking ibandronate who cannot remember when the last dose was. c. Client taking raloxifene who reports unilateral calf swelling. d. Client taking risedronate who reports occasional dyspepsia.

ANS: C The client on raloxifene needs to be assessed first because of the potential for deep vein thrombosis, which is an adverse effect of raloxifene. The client with flank pain may have had a kidney stone but is not acutely ill now. The client who cannot remember taking the last dose of ibandronate can be seen last. The client on risedronate may need to change medications.

A nurse teaches a client who is at risk for colorectal cancer. Which dietary recommendation would the nurse teach the client? a. "Eat low-fiber and low-residual foods." b. "White rice and bread are easier to digest." c. "Add vegetables such as broccoli and cauliflower to your diet." d. "Foods high in animal fat help to protect the intestinal mucosa."

ANS: C The client would be taught to modify his or her diet to decrease animal fat and refined carbohydrates. The client should also increase high-fiber foods and Brassica vegetables, including broccoli and cauliflower, which help to protect the intestinal mucosa from colon cancer.

The nurse is caring for a client who had a closed reduction of the left arm and notes a large wet area of drainage on the cast. What action is the most important? a. Cut off the old cast. b. Document the assessment. c. Notify the primary health care provider. d. Wrap the cast with gauze.

ANS: C The primary health care provider should be notified to examine the client and determine the source of the drainage. The nurse's assessment should be documented, but that is not the most important action.

After a total knee arthroplasty, a client is on the postoperative nursing unit with a continuous femoral nerve blockade. On assessment, the nurse notes the skin of both legs is pale pink, warm, and dry, but the client is unable to dorsiflex or plantarflex the surgical foot. What action would the nurse take next? a. Document the findings and monitor as prescribed. b. Increase the frequency of monitoring the client. c. Notify the surgeon or anesthesia provider immediately. d. Palpate the client's bladder or perform a bladder scan.

ANS: C With the femoral nerve block, the client would still be able to dorsiflex and plantarflex the affected surgical foot. Since this client has an abnormal finding, the nurse would notify either the surgeon or the anesthesia provider immediately. Documentation is the last priority. Increasing the frequency of assessment may be appropriate, but first the nurse must notify the appropriate provider. Palpating the bladder is not related.

A nurse is providing education to a community women's group about lifestyle changes helpful in preventing osteoporosis. What topics does the nurse cover? (Select all that apply.) a. Cut down on tobacco product use. b. Limit alcohol to two drinks a day. c. Strengthening exercises are important. d. Take recommended calcium and vitamin D. e. Walk for 30 minutes at least three times a week.

ANS: C, D, E Lifestyle changes can be made to decrease the occurrence of osteoporosis and include strengthening and weight-bearing exercises and getting the recommended amounts of both calcium and vitamin D. Tobacco should be totally avoided. Women should not have more than one drink per day.

The nurse reviews a list of drugs that can cause secondary osteoporosis. Which drugs are most commonly associated with this health problem? (Select all that apply.) a. Antianxiety agents b. Antibiotics c. Barbiturates d. Corticosteroids e. Loop diuretics

ANS: C, D, E Several classes of drugs can cause secondary osteoporosis, including barbiturates, corticosteroids, and loop diuretics. Antianxiety agents and antibiotics are not associated with the formation of osteoporosis.

The nurse is caring for a postoperative client who have a regional nerve blockade for a surgical tibial fracture repair this morning. What assessment finding would the nurse expect? a. Client reports nausea and vomiting. b. Client reports tingling in the surgical leg. c. Client responds well to imagery. d. Client reports little to no pain.

ANS: D A regional nerve blockade can last for about 24 hours so the client has little to no pain until it wears off. The blockade is localized and therefore does not cause nausea or vomiting.

The nurse interviews an older client with moderate osteoarthritis and her husband. What psychosocial assessment question would the nurse include? a. "Do you feel like hurting yourself or others?" b. "Are you planning to retire due to your disease?" c. "Do you ask your husband for assistance?" d. "Do you experience discomfort during sex?"

ANS: D Although some clients can become depressed and anxious as a result of having OA, suicidal ideation is not common. The nurse should not assume that an older adult will want to retire or that the client will need help from her husband. Many clients avoid sexual intercourse because of joint pain and stiffness.

The nurse assesses a client with rheumatoid arthritis (RA) and Sjögren syndrome. What assessment would be most important for this client? a. Abdominal assessment b. Oxygen saturation c. Breath sounds d. Visual acuity

ANS: D Sjögren syndrome may be seen in clients with RA and manifests with dryness of the eyes, mouth, and vagina in females. Visual disturbances can occur. The other assessments are not related to Sjögren syndrome.

After teaching a client with a fractured humerus, the nurse assesses the client's understanding. Which dietary choice demonstrates that the client correctly understands the nutrition needed to assist in healing the fracture? a. Baked fish with orange juice and a vitamin D supplement b. Bacon, lettuce, and tomato sandwich with a vitamin B supplement c. Vegetable lasagna with a green salad and a vitamin A supplement d. Roast beef with low-fat milk and a vitamin C supplement

ANS: D The client with a healing fracture needs supplements of vitamins B and C and a high-protein, high-calorie diet. Milk for calcium supplementation and vitamin C supplementation is appropriate. Meat would increase protein in the diet that is necessary for bone healing. Fish, a sandwich, and vegetable lasagna would provide less protein.

An older client who fell at home is admitted to the emergency department and reports pain in her left groin and behind her left knee. What action would the nurse anticipate? a. Administer IV push morphine. b. Prepare for application of a leg cast. c. Begin oxygen at 6 L/min via mask. d. Obtain a left hip x-ray.

ANS: D The location of the client's pain indicates a possible fractured hip and therefore an x-ray of the hip is needed. A leg cast is not appropriate and oxygen may not be needed. Medication to make the client more comfortable would likely be needed after a diagnosis is determined.

A nurse cares for a client who has a family history of colorectal cancer. The client states, "My father and my brother had colon cancer. What is the chance that I will get cancer?" How would the nurse respond? a. "If you eat a low-fat and low-fiber diet, your chances decrease significantly." b. "You are safe. This is an autosomal dominant disorder that skips generations." c. "Preemptive surgery and chemotherapy will remove cancer cells and prevent cancer." d. "You should have a colonoscopy more frequently to identify abnormal polyps early."

ANS: D The nurse would encourage the patient to have frequent colonoscopies to identify abnormal polyps and cancerous cells early. The abnormal gene associated with colon cancer is an autosomal dominant gene mutation that does not skip a generation and places the client at high risk for cancer. Changing the client's diet to more high-fiber (not low-fiber) and preemptive chemotherapy may decrease the client's risk of colon cancer but will not prevent it.

A client is scheduled to have a total hip arthroplasty. What preoperative teaching by the nurse is most important? a. Teach the need to discontinue all medications for 5 days before surgery. b. Teach the patient about foods high in protein, Vitamin C, and iron. c. Explain to the client the possible need for blood transfusions postoperatively. d. Remind the client to have all dental procedures completed at least 2 weeks prior to surgery.

ANS: D The nurse would include teaching about dental procedures to avoid infection after new joint has been inserted. Planned procedures would be completed at least 2 weeks before surgery and the client will need to tell any future primary health care providers about having a total joint arthroplasty. Only home medications prescribed that increase the risk for bleeding or clotting need to be discontinued 5 to 10 days before surgery. Clients need to be aware that any postoperative anemia may need to be treated with a blood transfusion, but it is not the most important. Diets high in protein, Vitamin C, and iron help with tissue repair, but are not the most important.

7. A nurse cares for a young client with a new ileostomy. The client states, "I cannot go to prom with an ostomy." How would the nurse respond? a. "Sure you can. Purchase a prom dress one size larger to hide the ostomy appliance." b. "The pouch won't be as noticeable if you avoid broccoli and carbonated drinks prior to the prom." c. "Let's talk to the ostomy nurse about options for ostomy supplies and dress styles." d. "You can remove the pouch from your ostomy appliance when you are at the prom so that it is less noticeable."

c. "Let's talk to the ostomy nurse about options for ostomy supplies and dress styles." The ostomy nurse is a valuable resource for patients, providing suggestions for supplies and methods to manage the ostomy. A larger dress size will not necessarily help hide the ostomy appliance. Avoiding broccoli and carbonated drinks does not offer reassurance for the client. Ileostomies have an almost constant liquid effluent, so pouch removal during the prom is not feasible.

A male client's sister was recently diagnosed with colorectal cancer (CRC), and his brother died of CRC 5 years ago. The client asks whether he will inherit the disease. How would the nurse respond? a. "Have you asked your primary health care provider about your chances ?" b. "It is hard to know what can predispose a person to develop a certain disease." c. "The only way to know whether you are predisposed to CRC is by genetic testing." d. "No. Just because they both had CRC doesn't mean that you will have it, too."

c. "The only way to know whether you are predisposed to CRC is by genetic testing." The nurse's best response to the client who asks if he will inherit CRC is "the only way to know whether you are predisposed to CRC is by genetic testing." Genetic testing is the only definitive way to determine whether the patient has a predisposition to develop CRC.

A nurse is caring for a client who is taking lubiprostone. The nurse should tell the client that lubiprostone can cause which of the following adverse drug reactions? A. Nausea B. Constipation C. Urinary retention D. Sore throat

A. Nausea

A nurse is caring for a client who has a prescription for alosetron. The nurse should recognize that alosetron therapy is effective when the client reports which of the following? A. One formed stool per day B. Urination without burning C. Cessation of nausea or vomiting D. Reduced GI reflux effects

A. One formed stool per day

A nurse assesses an older adult who was admitted 2 days ago with a fractured hip. The nurse notes that the client is confused and restless with an oxygen saturation of 88%. Which action would the nurse take first? a. Administer oxygen via nasal cannula. b. Re-position to a semi-Fowler position. c. Increase the intravenous flow rate. d. Assess response to pain medication.

ANS: A The client is at high risk for a fat embolism syndrome and pulmonary embolus. Although these complications are life-threatening emergencies, the nurse would administer oxygen first and then notify the primary health care provider. Oxygen administration can reduce the risk for cerebral damage from hypoxia. Pain medication most likely would not cause the client to be restless.

During an interview, the client tells the nurse that the client has a duodenal ulcer. Which assessment finding would the nurse expect? A: Hematemesis B: Pain when eating C: Melena D: Weight loss

C: Melena All of the other assessment findings are more commonly seen in clients who have gastric ulcers rather than duodenal ulcers.

The nurse is teaching a client about risk factors for esophageal cancer. Which risk factors would the nurse include? (Select all that apply.) a. Alcohol intake b. Obesity c. Smoking d. Lack of fresh fruits and vegetables e. Untreated GERD f. Use of NSAIDs

ANS: A, B, C, D, E All of these factors increase the risk of esophageal cancer except for the use of NSAIDs. Untreated GERD causes damage to esophageal tissue which may develop into Barrett esophagus, or precancerous cells.

A client who had a recent total knee arthroplasty will be using a continuous passive motion (CPM) machine after discharge at home. What health teaching about the CPM machine will the nurse include? (Select all that apply.) a. "Keep the machine padded well to prevent skin breakdown." b. "Ensure that your leg is placed properly on the machine." c. "Use the machine as prescribed but not at mealtime." d. "When the machine is not being used, do not store it on the floor." e. "Check that the cycle and range of motion is kept at the level prescribed."

ANS: A, B, C, D, E Although not used as often today, some clients are prescribed to use the CPM machine to increase range of motion in the surgical knee. All of these teaching points are important for any client who uses a CPM machine.

The nurse is caring for a client diagnosed with probable gastroesophageal reflux disease (GERD). What assessment finding(s) would the nurse expect? (Select all that apply.) a. Dyspepsia b. Regurgitation c. Belching d. Coughing e. Chest discomfort f. Dysphagia

ANS: A, B, C, D, E, F All of these signs and symptoms are commonly seen in clients who have GERD.

The nurse is caring for a client who recently sustained a sports injury to his right leg. What nursing interventions are appropriate for this client? (Select all that apply.) a. Immobilize the right leg. b. Apply heat immediately after the injury. c. Use compression to support the leg. d. Obtain an x-ray to detect possible fracture. e. Elevate the right leg to decrease swelling. f. Administer an opioid every 4 to 6 hours.

ANS: A, C, D, E The client who experiences a sports injury should be managed using the RICE treatment plan. Rest, ice, compression, and elevation are all appropriate. Heat would increase swelling and probably pain. An x-ray would be obtained to determine if one or more fractures are present. Opioids may not be needed depending on the nature of the injury.

A nurse knows that job-related risks for developing oral cancer include which occupations? (Select all that apply.) a. Coal miner b. Electrician c. Metal worker d. Plumber e. Textile worker

ANS: A, C, D, E The occupations of coal mining, metal working, plumbing, and textile work produce exposure to polycyclic aromatic hydrocarbons (PAHs), which are known carcinogens. Electricians do not have this risk.

A client who had a fractured ankle open reduction internal fixation (ORIF) 4 weeks ago reports burning pain and tingling in the affected foot. For which potential complication would the nurse anticipate? a. Delayed bone healing b. Complex regional pain syndrome c. Peripheral neuropathy d. Compartment syndrome

ANS: B Burning pain and tingling that occurs weeks or months after a fracture or other trauma may indicate complex regional pain syndrome. Compartment syndrome tends to occur within days of the initial injury.

A client has been prescribed denosumab. What health teaching about this drug is most appropriate for the nurse to include? a. "Drink at least 8 ounces (240 mL) of water with it." b. "Make appointments to come get your injection." c. "Sit upright for 30 to 60 minutes after taking it." d. "Take the drug on an empty stomach."

ANS: B Denosumab is given by subcutaneous injection twice a year. The client does not need to drink 8 ounces (240 mL) of water with this medication as it is not taken orally. The client does not need to remain upright for 30 to 60 minutes after taking this medication, nor does the client need to take the drug on an empty stomach.

A nurse is caring for four clients. After the hand-off report, which client would the nurse see first? a. Client with osteoporosis and a white blood cell count of 27,000/mm3(27 x 10^9/L) b. Client with osteoporosis and a bone fracture who requests pain medication c. Post-microvascular bone transfer client whose distal leg is cool and pale d. Client with suspected bone tumor who just returned from having a spinal CT

ANS: C This client is the priority because the assessment findings indicate a critical lack of perfusion. A high white blood cell count is an expected finding for the client with osteoporosis. The client requesting pain medication should be seen second. The client who just returned from a CT scan is stable and needs no specific postprocedure care.

A nurse assesses a client with a pelvic fracture. Which assessment finding would the nurse identify as a complication of this injury? a. Hypertension b. Diarrhea c. Infection d. Hematuria

ANS: D The pelvis is very vascular and close to major organs. Injury to the pelvis can cause integral damage that may manifest as blood in the urine (hematuria) or stool. The nurse would also assess for signs of hemorrhage and hypovolemic shock, which include hypotension and tachycardia. Diarrhea and infection are not common complications of a pelvic fracture.

A nurse plans care for a client who has an external fixator on the lower leg. Which intervention would the nurse include in the plan of care to decrease the client's risk for infection? a. Washing the frame of the fixator once a day b. Releasing fixator tension for 30 minutes twice a day c. Avoiding moving the extremity by holding the fixator d. Scheduling for pin care to be provided every shift

ANS: D To decrease the risk for infection in a client with skeletal traction or external fixation, the nurse would provide routine pin care and assess for signs and symptoms of infection at the pin sites every shift.

A nurse teaches assistive personnel (AP) about providing hygiene for a client in traction. Which statement would the nurse include as part of the teaching about this client's care? a. "Remove the traction when re-positioning the client." b. "Assess the client's skin when performing a bed bath." c. "Provide pin care by using alcohol wipes to clean the sites." d. "Ensure that the weights remain freely hanging at all times."

ANS: D Traction weights should be freely hanging at all times. They should not be lifted manually or allowed to rest on the floor. The client should remain in traction during hygiene activities. The nurse would assess the client's skin and provide pin and wound care for a patient who is in traction; this would not be delegated to the AP.

The nurse is teaching a client with a newly created colostomy about foods to limit or avoid because of flatulence or odors. Which foods should be avoided? (Select all that apply.) Select all that apply. a. Mushrooms b. Peas c. Onions d. Broccoli e. Buttermilk f. Yogurt

a. Peas b. Onions c. Broccoli Foods the patient with a newly created colostomy needs to limit or avoid because of flatulence or odors include: broccoli, mushrooms, onions, and peas. Buttermilk will help prevent odors. Yogurt can help prevent flatus.

The nurse is teaching a group of clients with irritable bowel syndrome (IBS) about complementary and alternative therapies. What does the nurse suggest as possible treatment modalities? (Select all that apply.) Select all that apply. a. Yoga b. Acupuncture c. Peppermint oil capsules d. Decreasing physical activities e. Meditation

a. Yoga b. Acupuncture c. meditation Possible treatment modalities the nurse suggests for a client with IBS include: acupuncture, meditation, peppermint oil capsules, and yoga. Acupuncture is recommended as a complementary therapy for IBS. Meditation, yoga, and other relaxation techniques help many patients manage stress and their IBS symptoms. Research has shown that peppermint oil capsules may be effective in reducing symptoms of IBS. Regular exercise is important for managing stress and promoting bowel elimination.

The home health nurse is teaching a client about the care of a new colostomy. Which statement by the client demonstrates a correct understanding of the health teaching? a. "If the skin around the stoma is red or scratched, it will heal soon." b. "I need to strive for a very tight fit when applying the barrier around the stoma." c. "A dark or purplish-looking stoma is normal and would not concern me." d. "I need to check for leakage underneath my colostomy."

d. "I need to check for leakage underneath my colostomy." The client's statement, "I need to check for leakage underneath my colostomy" shows that the patient correctly understands the instructions about how to care for a new colostomy. The pouch system must be checked frequently for evidence of leakage to prevent excoriation.A purplish stoma is indicative of ischemia and necrosis. Redness or scratched skin around the stoma must be reported to prevent it from beginning to break down. An overly tight fit may lead to necrosis of the stoma.

A nurse should recognize that misoprostol is contraindicated for a client who has which of the following conditions? A. A seizure disorder B. Rheumatoid arthritis C. A positive pregnancy test D. Heart failure

C. A positive pregnancy test

Which of the following is (are) (a) risk factor(s) for gastric cancer? (Select all that apply.) A: Achlorhydria B: Chronic atrophic gastritis C: H. pylori infection D: Iron deficiency anemia E: Pernicious anemia

A: Achlorhydria B: Chronic atrophic gastritis C: H. pylori infection E: Pernicious anemia Achlorhydria, chronic atrophic gastritis, H. pylori infection, and pernicious anemia are all risk factors for developing gastric cancer. Iron deficiency anemia is not a risk factor.

What action(s) by the nurse is (are) appropriate to promote nutrition in a client who had a partial gastrectomy? (Select all that apply.) A: Administer vitamin B12 injections. B: Ask the primary health care provider about folic acid replacement. C: Educate the client on enteral feedings. D: Obtain consent for total parenteral nutrition. E: Provide iron supplements for the client.

A: Administer vitamin B12 injections. B: Ask the primary health care provider about folic acid replacement. E: Provide iron supplements for the client. After a partial or total gastrectomy, clients are at high risk for anemia due to vitamin B12 deficiency, folic acid deficiency, or iron deficiency. The nurse would provide supplements for all these nutrients. The client does not need enteral feeding or total parenteral nutrition.

A client has dumping syndrome. What menu selections indicate the client understands the correct diet to manage this condition? (Select all that apply.) A: Apricots B: Coffee cake C: Milk shake D: Potato soup E: Steamed broccoli

A: Apricots D: Potato soup Canned apricots and potato soup are appropriate selections as they are part of a high-protein, high-fat, and low- to moderate-carbohydrate diet. Coffee cake and other sweets must be avoided. Milk products and sweet drinks such as shakes must be avoided. Gas-forming foods such as broccoli must also be avoided.

A client has dumping syndrome after a partial gastrectomy. Which action by the nurse would be appropriate? A: Arrange a dietary consult. B: Increase fluid intake. C: Limit the client's foods. D: Make the client NPO.

A: Arrange a dietary consult. The client with dumping syndrome after a gastrectomy has multiple dietary needs. A referral to the registered dietitian nutritionist will be extremely helpful. Food and fluid intake is complicated and needs planning. The client should not be NPO.

The nurse is caring for a client who has frequent gastric pain and dyspepsia. Which procedure would the nurse expect for the client to make an accurate diagnosis? A: Esophagogastroduodenoscopy (EGD) B: Abdominal arteriogram C: Nuclear medicine scan D: Magnetic resonance imaging (MRI)

A: Esophagogastroduodenoscopy (EGD) The gold standard for diagnosing disorders of the stomach is an EGD which allows direct visualization by the endoscopist into the esophagus, stomach, and duodenum.

1.5. A nurse prepares to discharge a client who is newly diagnosed with a chronic inflammatory bowel disease. Which questions would the nurse ask in preparation for discharge? (Select all that apply.) a. Does your gym provide yoga classes? b. When should you contact your provider? c. What do you plan to eat for dinner? d. Do you have a scale for daily weights? e. How many bathrooms are in your home?

ANS: A, B, C, E a. Does your gym provide yoga classes? b. When should you contact your provider? c. What do you plan to eat for dinner? e. How many bathrooms are in your home? A home assessment for a client who has a chronic inflammatory bowel disease would include identifying adequacy and availability of bathroom facilities, opportunities for rest and relaxation, and the client's knowledge of dietary therapy, and when to contact the primary health care provider. The client does not need to perform daily weights.

The Certified Wound, Ostomy, and Continence Nurse (CWOCN) is teaching a client with colorectal cancer how to care for a newly created colostomy. Which statement by the client indicates a correct understanding of the necessary self-management skills? a. "If I have any leakage, I'll put a towel over it." b. "I can put aspirin tablets in the pouch in order to reduce odor" c. "I will apply a nonalcoholic skin sealant and let it dry before applying the bag." d. "I will have my spouse change the bag for me."

c. "I will apply a nonalcoholic skin sealant and let it dry before applying the bag." The nurse would teach the client and family to apply a skin sealant (preferably without alcohol) and allow it to dry before application of the appliance (colostomy bag) to facilitate less painful removal of the tape or adhesive. It is not realistic that the spouse will always change the patient's bag and does not reflect correct understanding of self-management skills. A towel is not an acceptable or effective way to cope with leakage. Putting an aspirin in the pouch will not reduce odor and can lead to ulcers in the stoma.offers reassurance and is a "pat" statement, making it nontherapeutic. "Why" questions place patients on the defense and are not therapeutic because they close the conversation.

11. A nurse assesses a client who has ulcerative colitis and severe diarrhea. Which assessment would the nurse complete first? a. Inspection of oral mucosa b. Recent dietary intake c. Heart rate and rhythm d. Percussion of abdomen

c. Heart rate and rhythm Although the client with severe diarrhea may experience skin irritation and hypovolemia, the client is most at risk for cardiac dysrhythmias secondary to potassium and magnesium loss from severe diarrhea. The client would have her or his electrolyte levels monitored, and electrolyte replacement may be necessary. Oral mucosa inspection, recent dietary intake, and abdominal percussion are important parts of physical assessment but are lower priority for this patient than heart rate and rhythm.

A nurse is caring for a client who is taking allopurinol to treat gout and has a new prescription for azathioprine to treat ulcerative colitis. For which of the following reasons should the nurse clarify these prescriptions with the provider? A. Allopurinol delays the conversion of azathioprine and can lead to toxicity. B. Azathioprine increases the effectiveness of allopurinol. C. Allopurinol increases the metabolism of azathioprine and can require an increased dosage. D. Azathioprine decreases the effectiveness of allopurinol.

A. Allopurinol delays the conversion of azathioprine and can lead to toxicity.

A nurse is assessing a client who was administered ondansetron IV 1 hour ago. Which of the following findings should the nurse recognize as an adverse effect of this drug? A. Dizziness B. Bradycardia C. Tardive dyskinesia D. Dyspepsia

A. Dizziness

A nurse is teaching a client who has a new prescription for methotrexate. The nurse should instruct the client to monitor for manifestations of which of the following conditions? A. Gout B. Constipation C. Insomnia D. Hirsutism

A. Gout

A nurse should recognize that diphenoxylate/atropine should be used with caution for a client who has which of the following conditions? A. Inflammatory bowel disease B. Thrombophlebitis C. Agranulocytosis D. Immunization with a live virus vaccine within the last 6 weeks

A. Inflammatory bowel disease

A nurse is caring for an older adult client who has renal impairment and a new prescription for cimetidine. The nurse should instruct the client to report which of the following manifestations? A. Lethargy B. Cellulitis C. Dry mouth D. Myalgia

A. Lethargy

A nurse is caring for a male client who asks the nurse about taking alosetron for irritable bowel syndrome with diarrhea (IBS-D) lasting 3 months. Which of the following information should the nurse provide the client about alosetron? A. The drug is prescribed to female clients who have IBS-D lasting more than 6 months. B. The drug is prescribed to clients who have chronic diarrhea lasting more than 12 months. C. The drug is most beneficial for male clients who have inflammatory bowel disease. D. The drug is most beneficial in treating clients who have constipation-predominant IBS (IBS-C).

A. The drug is prescribed to female clients who have IBS-D lasting more than 6 months.

A nurse assesses a client with irritable bowel syndrome (IBS). Which questions would the nurse include in this client's assessment? (Select all that apply.) a. "Which food types cause an exacerbation of symptoms?" b. "Where is your pain or discomfort and what does it feel like?" c. "Have you lost a significant amount of weight lately?" d. "Are your stools soft, watery, and black?" e. "Do you often experience nausea and vomiting"

ANS: A, B The nurse would ask the client about factors that may cause exacerbations of symptoms, including food, stress, and anxiety. The nurse would also assess the location, intensity, and quality of the patient's pain or discomfort. Clients who have IBS do not usually lose weight, have nausea and vomiting, or have stools that are black.

1.2. The nurse assesses a client with ulcerative colitis. Which complications are paired correctly with their physiologic processes? (Select all that apply.) a. Lower gastrointestinal bleeding—erosion of the bowel wall b. Abscess formation—localized pockets of infection develop in the ulcerated bowel lining c. Toxic megacolon—transmural inflammation resulting in pyuria and fecaluria d. Nonmechanical bowel obstruction—paralysis of colon resulting from colorectal cancer e. Fistula—dilation and colonic ileus caused by paralysis of the colon

ANS: A, B, D a. Lower gastrointestinal bleeding—erosion of the bowel wall b. Abscess formation—localized pockets of infection develop in the ulcerated bowel lining d. Nonmechanical bowel obstruction—paralysis of colon resulting from colorectal cancer Lower GI bleeding can lead to erosion of the bowel wall. Abscesses are localized pockets of infection that develop in the ulcerated bowel lining. Nonmechanical bowel obstruction is paralysis of the colon that results from colorectal cancer. When the inflammation is transmural, fistulas can occur between the bowel and bladder resulting in pyuria and fecaluria. Paralysis of the colon causing dilation and subsequent colonic ileus is known as a toxic megacolon.

After teaching a client with irritable bowel syndrome (IBS), a nurse assesses the client's understanding. Which menu selection indicates that the client correctly understands the dietary teaching? a. Ham sandwich on white bread, cup of applesauce, carbonated beverage b. Broiled chicken with brown rice, steamed broccoli, glass of apple juice c. Grilled cheese sandwich, small banana, cup of hot tea with lemon d. Baked tilapia, fresh green beans, cup of coffee with low-fat milk

ANS: B Patients with IBS are advised to eat a high-fiber diet. Chicken with brown rice, broccoli, and apple juice has the highest fiber content. Clients should avoid alcohol, caffeine, and other gastric irritants.

A client has a recurrence of gastric cancer and is crying. What response by the nurse is most appropriate? A: "Do you have family or friends for support?" B: "Would you tell me what you are feeling now." C: "Well, we knew this would probably happen." D: "Would you like me to refer you to hospice?"

B: "Would you tell me what you are feeling now." The nurse assesses the client's emotional state with open-ended questions and statements and shows a willingness to listen to the client's concerns. Asking about support people is very limited in nature, and "yes-or-no" questions are not therapeutic. Stating that this was expected dismisses the client's concerns. The client may or may not be ready to hear about hospice, and this is another limited, yes-or-no question.

The nurse is caring for a client with a long history of peptic ulcer disease. What assessment findings would the nurse anticipate if the client experiences upper gastrointestinal (GI) bleeding? (Select all that apply.) A: Decreased heart rate B: Decreased blood pressure C: Bounding radial pulse D: Dizziness E: Hematemesis F: Decreased urinary output

B: Decreased blood pressure D: Dizziness E: Hematemesis F: Decreased urinary output The client who has upper GI bleeding would likely have vomiting that contains blood (hematemesis), and would have signs and symptoms of dehydration such as a decreased blood pressure, dizziness, and/or decreased urinary output. The heart rate increases rather than decreases and the pulse is weak rather than bounding in clients who are dehydrated.

A client who had a partial gastrectomy 3 days ago begins to experience vertigo, sweating, and tachycardia about 30 minutes after eating breakfast. What postoperative complication would the nurse suspect? A: Pyloric obstruction B: Dumping syndrome C: Delayed gastric emptying D: Pernicious anemia

B: Dumping syndrome Dumping syndrome causes autonomic symptoms as food quickly leaves the stomach due to its decreased size after surgery.

A client has a nasogastric (NG) tube as a result of an upper gastrointestinal (GI) hemorrhage. What comfort measure would the nurse remind assistive personnel (AP) to provide? A: Lavaging the tube with ice water B: Performing frequent oral care C: Re-positioning the tube every 4 hours D: Taking and recording vital signs

B: Performing frequent oral care Clients with NG tubes need frequent oral care both for comfort and to prevent infection. Lavaging the tube is done by the nurse. Repositioning the tube, if needed, is also done by the nurse. The can take vital signs, but this is not a comfort measure.

A nurse is providing teaching to a client who is about to start taking psyllium to treat constipation. Which of the following instructions should the nurse include? (Select all that apply). A. Expect results in 6-12 hours B. Urinate every 4 hours C. Take the drug with at least 8 oz (237 mL) of fluid D. Avoid activities that require alertness E. Increase fluid and fiber intake

C, E

A client with peptic ulcer disease is in the emergency department and reports gastric pain that has gotten much worse over the last 24 hours. The client's blood pressure when lying down is 112/68 mm Hg and when standing is 98/52 mm Hg. What action by the nurse is most appropriate? A: Administer a proton pump inhibitor (PPI). B: Call the Rapid Response Team. C: Start a large-bore IV with normal saline. D: Tell the patient to remain lying down.

C: Start a large-bore IV with normal saline. This client has orthostatic changes to the blood pressure, indicating fluid volume loss. The nurse would start a large-bore IV with isotonic solution. PPIs are not a treatment for an ulcer. The Rapid Response Team is not needed at this point. The client should be put on safety precautions, which includes staying in bed, but this is not the most appropriate action at this time.

A nurse is teaching a client who recently had a myocardial infarction and has a new prescription for docusate sodium. The nurse should inform the client that docusate sodium has which of the following therapeutic effects? A. Reduces inflammation B. Reduces gastric acid C. Prevents diarrhea D. Prevents straining

D. Prevents straining

17. A nurse cares for a client with a new ileostomy. The client states, "I don't think my friends will accept me with this ostomy." How would the nurse respond? a. "Your friends will be happy that you are alive." b. "Tell me more about your concerns." c. "A therapist can help you resolve your concerns." d. "With time you will accept your new body."

b. "Tell me more about your concerns." Social anxiety and apprehension are common in clients with a new ileostomy. The nurse would encourage the client to discuss concerns by restating them in an open-ended manner. The nurse would not minimize the client's concerns or provide false reassurance.

The nurse is providing teaching on ways to promote bowel health and disease prevention. Which statement will the nurse include in this teaching? a. "You should start colorectal cancer screening when you are over 70 years of age." b. "You only need to have regular colonoscopies if there is colorectal cancer in your family.' c. "If you perform fecal occult blood tests every 5 years, you don't need a colonoscopy." d. "You should have a colonoscopy every 10 years starting at 45 years of age."

d. "You should have a colonoscopy every 10 years starting at 45 years of age." The American Cancer Society recommends that for individuals of average risk for colorectal cancer (CRC), a colonoscopy every 10 years or flexible sigmoidoscopy every 5 years is adequate. The screening should begin for adults of 45 years of age or older unless individuals are at high risk for CRC.

A client with an intestinal obstruction has pain that changes from a "colicky" intermittent type to constant discomfort. After a complete assessment, what action would the nurse plan implement at this time? a. Change the nasogastric suction level from "intermittent" to "continuous." b. Administer medication for pain based on the client's pain level. c. Position the client in a semi- or high-Fowler position. d. Prepare the client for emergency surgery in collaboration with the health team.

d. Prepare the client for emergency surgery in collaboration with the health team. The appropriate nursing action for a client with intestinal obstruction whose pain changes from "colicky" intermittent type to constant discomfort is to prepare surgery because this change is most likely indicative of perforation or peritonitis and will require immediate surgical intervention.Pain medication may mask the client's symptoms but will not address the root cause. A change in the nasogastric suction rate will not resolve the cause of the client's pain and could be particularly ineffective if a nonvented tube is in use.


Kaugnay na mga set ng pag-aaral

60 - The Second World War, 1942-1945 (comprehensive)

View Set

Tennessee Insurance Laws and Rules (Core)

View Set

Positive and Negative Interactions Between Europeans and Native Americans

View Set